Answer and explantion?​

Answers

Answer 1

Answer:

x = 134

Step-by-step explanation:

All right, I'm not the best at explaining these but I will try. So first these two are parallel lines (Because you can see the 2 arrows >> like this). When looking at the problem what I would first to is find the angle on the other side of 46, which is 134. Since these are parallel lines the line up there will also equal the same. So x would equal 134 and the angle on the other side will be 46. I hope that makes sense, if not you can ask me what doesn't make sense and I will try to explain it

Answer 2

Answer:

x = 134°

Step-by-step explanation:

x and 46° are same- side interior angles and sum to 180° , so

x + 46° = 180° ( subtract 46° from both sides )

x = 134°


Related Questions

Find the value of x rounded to the nearest tenth.

Answers

I cannot see whole picture .. although I would say 2.8

Simplify. (x+y)/(x^2y)-(x-2y)/(xy^2)

Answers

Math Topic : Algebraic Simplification

Answer:

[tex]{ \tt{ = \frac{(x + y)}{ {x}^{2}y } - \frac{(x - 2y)}{ {xy}^{2} } }} [/tex]

Find the LCM of denominators: x²y²

[tex]{ \tt{ = \frac{y(x + y) - x(x - 2y)}{ {x}^{2} {y}^{2} } }} \\ \\ = { \tt{ \frac{xy + {y}^{2} - {x}^{2} +2xy }{ {x}^{2} {y}^{2} } }}[/tex]

Simplify further:

[tex] = { \tt{ \frac{(y - x)(y + x) +3xy}{ {(xy)}^{2} } }} \\ \\ = { \tt{ \frac{(y - x)(y + x)}{ {(xy)}^{2} } - \frac{3}{xy} }}[/tex]

solve please 14a⁹b-8a³d÷ 2a³​

Answers

Answer:

7a^6b-4d

Step-by-step explanation:

[tex]\frac{14a^9b - 8a^3d}{2a^3} \\\frac{2a^3(7a^6b - 4d)}{2a^3} \\\\7a^6b-4d[/tex]

The Tres Difficult race helps raise money for charity. According to the website, of the proceeds from ticket sales go directly to charity.
2/5
Last year they made $8000 from ticket sales. How much was given to charity?

Answers

Answer:

3200

Step-by-step explanation:

We need to find 2/5 of the tickets sales

2/5 * 8000

3200

Answer:

3200

Step-by-step explanation:

you need to find what 2/5 is and the you take that away from 8000 and then you have your answer of 3200

equation for perpendicular to the line -7x + 3y = -10j contains the point (-2,-4)

Answers

Answer:

y = 7/3x + 2/3

Step-by-step explanation:

-7x + 3y = -10

3y = 7x - 10

y = 7/3x - 10/3

-4 = 7/3(-2) + b

-4 = -14/3 + b

2/3 = b

Estimate the square root between two consecutive whole numbers of sqrt [55]

Answers

9514 1404 393

Answer:

  7.4 . . . . between 7 and 8

Step-by-step explanation:

55 is between the perfect squares 49 = 7² and 64 = 8². Using linear interpolation, the square root is approximately 7 +(55-49)/(64-49) = 7 6/15 = 7.4

√55 ≈ 7.4 . . . . approximate root by linear interpolation

_____

Additional comment

A way to improve the estimate of the root is to use the "Babylonian method" of iterating the root. Divide the original number (55) by the estimate of the root, and average that result with the estimate:

  next best guess = (55/7.4 +7.4)/2 = 7 77/185 ≈ 7.4162_162(repeating)

This matches the actual root when rounded to 4 decimal places. The number of accurate decimal places approximately doubles with each iteration.

__

Another way to improve the estimate is to modify the fractional portion. (The above method converges on a root more quickly.) For this, the iteration of the fractional part of the root is ...

  next fractional part = 6/(14 +(fractional part))

where 6/14 is the linear estimate fractional value with 1 subtracted from its denominator.

For one iteration, the new estimate of the fractional part is 6/(14 +6/15) = 5/12, so the root estimate is about 7.4167 compared to the above 7.4162.

During​ spring, young​ moose, unfamiliar with roads and​ traffic, are wandering around at night in a​ province, causing risk and road accidents. Suppose that the average number of road accidents involving moose was per day. The government increased the number of hunting licenses and cleared brush to improve​ drivers' visibility. On one day after these measures were​ implemented, there were road accidents involving moose. ​

Required:
a. What would be the chance of such accidents or​ fewer, assuming the​ government's measures were​ ineffective? ​
b. Do you think the​ government's measures were​ effective? State your reasons clearly.

Answers

Answer: A not b
Because it’s just not

Aiden is a spice trader. He sells any amount of cumin seeds from 1 kilogram to 1000 kilograms. He charges $5 for 1 kilogram and $2000 for 1000 kilograms.

p(w) models the price (in dollars) of w kilograms of cumin seeds in Aiden's shop.

Which number is more appropriate for the domain of p?
Choose 1 answer:

A. Integers
B. Real Numbers

What's the appropriate domain?
Choose 1 answer:

A. 5 ≤ w ≤ 1000
B. 5 ≤ w ≤ 2000
C. 1 ≤ w ≤ 1000
D. 1 ≤ w ≤ 2000

Answers

Answer:

A and B

Step-by-step explanation:

Domain of P should be Integers and it should vary from 1 to 2000

The number is more appropriate for the domain of 'p' is real numbers.

The appropriate domain can be represented as (1 ≤ w ≤ 1000).

What is the domain of a function?

"The domain of a function is the set of inputs accepted by the function."

Given, Aiden sells any amount of cumin seeds from 1 kilogram to 1000 kilograms.

He charges $5 for 1 kilogram and $2000 for 1000 kilograms.

w = the weight of the cumin seeds

Therefore, it can be represented as (1 ≤ w ≤ 1000).

p(w) = the price of w kg cumin seeds.

It can be represented as (5 ≤ p(w) ≤ 2000).

This domain is of real numbers.

Learn more about the domain of a function here: https://brainly.com/question/17354444

#SPJ3

pls help i’m dying i don’t know how to do this

Answers

Answer:

the answer that I got is 1

Answer: hi "1" is right, i checked it again ;)

Consider the probability that no more than 76 out of 504 computers will crash in a day. Choose the best description of the area under the normal curve that would be used to approximate binomial probability.

a. Area to the right of 75.5
b. Area to the right of 76.5
c. Area to the left of 75.5
d. Area to the left of 76.5
e. Area between 75.5 and 76.5

Answers

Answer:

e

Step-by-step explanation:

(A) Over 1000 students organized to celebrate running water and electricity. To count the exact number of students protesting, the chief organizer lined the students up in columns of different length. If the students are arranged in columns of 3, 5, and 7, then 2, 3, and 4 people are left out, respectively. What is the minimum number of students present? Solve it with Chinese Remainder Theorem. (B) Prove that for n> 1, if 935 = 5 x 11 x 17 divides n80 – 1, then 5, 11, and 17 do not divide n.

Answers

Solution :

A). x = 2 (mod 3)        [tex]$\mu = 3\times 5 \times 7 = 105$[/tex]

x = 3 (mod 5)       [tex]$y_1=35^{-1} (\mod 3)$[/tex]

x = 4 (mod 7)        [tex]y_1=2[/tex]

[tex]$y_2=21^{-1}(\mod5) = 1$[/tex]

[tex]$y_3=15^{-1}(\mod7) = 1$[/tex]

[tex]$x=2 \times 35 \times 2 + 3\times 21\times 1+4\times 15\times 1$[/tex]

   [tex]=140+63+60[/tex]

   [tex]=263[/tex]

   ≡ 53(mod 105)

Hence the solution is 105 k + 53 > 1000 for k = 10

Therefore, the minimum number of students = 1103

B). [tex]$\phi (935) = 640$[/tex]

By  Eulu's theory

[tex]$935 | a^{640}_n -1$[/tex] if n and 935 are coprime.

Now, [tex]$935|n^{80}-1$[/tex]  and 80 x 8 = 640

[tex]$935|n^{640}-1$[/tex]   ⇒  g(n,935) = 1

                     ⇒ 5, 11, 17 do not divide n

If you apply the changes below to the cube root parent function, F(x) = 3/x
what is the equation of the new function?
• Translate 1 unit right.
• Translate 1 unit up.
A. G(x) = 3/x-1+1
B. G(x) = 3/x +1-1
C. G(x) =3/ x - 1-1
D. G(x) = 3/x+1+1

Answers

9514 1404 393

Answer:

  A.  G(x) = ∛(x -1) +1

Step-by-step explanation:

The transformation f(x-h) +k represents a translation (right, up) by (h, k) of the parent function f(x).

Your translation of f(x) = ∛x by (1, 1) will give you the function ...

  G(x) = ∛(x -1) +1

Drag each tile to the correct box.
Match each equation with its solution.
n =
= -1
n = -25
n = 1
Equation
Solution
12 + 15 = -10
>
-511 = 1
- 13 = -12

Answers

Answer:

n = 1

n = - 1

n = - 1/5

n = - 25

Step-by-step explanation:

We are to obtain the value if n in the given equations :

1.)

n - 13 = - 12

To find, n ;

Add 13 to both sides

n - 13 + 13 = - 12 + 13

n = 1

2.)

n/5 = - 1/5

Multiply both sides by 5

n/5 * 5 = - 1/5 * 5

n = - 1

3.)

-5n = 1

Divide both sides by - 5

-5n/-5 = 1/-5

n = - 1/5

4.)

n + 15 = - 10

Subtract 15 from both sides :

n + 15 - 15 = - 10 - 15

n = - 25

A researcher wants to know if calcium is an effective treatment for lowering blood pressure. He assigns one randomly chosen group of volunteers to take calcium supplements; the other group will get placebo. At the end of the treatment period, he measures the difference in blood pressure. The 50 members of the calcium group had blood pressure lowered by an average of 25 points with a standard deviation of 10 points. The 50 members of the placebo group had blood pressure lowered by an average of 15 points with a standard deviation of 8 points. To analyze this information we will use a

Answers

Answer:

Two sample t procedure

Step-by-step explanation:

The two sample t test is used when want to test for equality between two population means. It tests whether the means of the two groups are equal or not equal.

We use this to analyse this information in this question because we do not have data available for the population standard deviation. Also we are to test for the significant difference between the two different groups of participants

dp/dt = t²p − p + t2 − 1.

Answers

dp/dt = t ² p - p + t ² - 1

Factorize the right side:

dp/dt = p (t ² - 1) + (t ² - 1)

dp/dt = (p + 1) (t ² - 1)

So the differential equation is separable as

dp/(p + 1) = (t ² - 1) dt

Integrate both sides:

∫ dp/(p + 1) = ∫ (t ² - 1) dt

ln|p + 1| = t ³/3 - t + C

Solve for p :

p + 1 = exp(t ³/3 - t + C )

p + 1 = C exp(t ³/3 - t )

p = C exp(t ³/3 - t ) - 1

PLEASE HELP with proving lines parallel

Answers

90 mins later we had to go to sleep for sub for two other people that were not

HELP PLEASE, Function problem

Answers

Answer:

-2

-1

-2

Step-by-step explanation:

please forgive me, but again, this is the simplest of the simplest things. how is that a problem ?

this costs so much more time to just put it in here and then copy answers than just doing it. this is literally a matter of seconds.

the functional value is -2 for all x that are not equal to 2.

and the functional value is -1, when x = 2

so, what is the problem ?

please see my other answer for more details on the solution.

please help me!!!!!!!!!!!!​

Answers

Step-by-step explanation:

24. = 249030/30

=8,301 rs

Answer:

24. 8301, divide 249030 by 30

25. 9989001, but i dont know the property

Step-by-step explanation:

Find the intercepts of the function y = 3x + 9

Answers

Step-by-step explanation:

To solve for the x-intercept, set y=0 then solve for x.

y=−3x−9. 0=−3x−9. 3x=−9.

x=−3 when y=0.

To solve for the y-intercept, set x=0 then solve for y.

y=−3x−9. y=−3(0)−9. y=−9 when x=0.

Hi there!

Y-intercept:

Set the x value to 0:

y = 3(0) + 9

y = 9 --> (0, 9)

X-intercept:

Set the y value to 0:

0 = 3x + 9

Solve for x:

-9 = 3x

x = -3 --> (-3, 0)

given a∥b and c∥d, m<4=35. find m<1, m<2, and m<3
its the rsm problem lesson 6 homework geometry

Answers

Answer:

<2 = 35degrees

<3 = 55degrees

Step-by-step explanation:

Find the diagram attached:

From the diagram:

<3 + <4 = 90

<3 + 35 = 90

<3 = 90 - 35

<3 = 55degrees

Also m<1 + m<2+ m<3 = 180 (sum of angles in a traingle)

90+<2+55=180

145+<2 = 180

<2 = 180 - 145

<2 = 35degrees

An ecologist finds 220 yellow-flowered plants and 180 white-flowered plants. Use the normal distribution to find the Lower boundary of a 95% confidence interval for the proportion of yellow-flowered plants. Which of the following answers is correct to 2 decimal places?
a. Lower boundary = 0.30
b. Lower boundary = 0.60
c. Lower boundary = 0.50
d. Lower boundary = 0.40

Answers

Answer:

c. Lower boundary = 0.50

Step-by-step explanation:

In a sample with a number n of people surveyed with a probability of a success of [tex]\pi[/tex], and a confidence level of [tex]1-\alpha[/tex], we have the following confidence interval of proportions.

[tex]\pi \pm z\sqrt{\frac{\pi(1-\pi)}{n}}[/tex]

In which

z is the z-score that has a p-value of [tex]1 - \frac{\alpha}{2}[/tex].

An ecologist finds 220 yellow-flowered plants and 180 white-flowered plants.

220 out of 220 + 180 = 400. So

[tex]n = 400, \pi = \frac{220}{400} = 0.55[/tex]

95% confidence level

So [tex]\alpha = 0.05[/tex], z is the value of Z that has a p-value of [tex]1 - \frac{0.05}{2} = 0.975[/tex], so [tex]Z = 1.96[/tex].  

The lower limit of this interval is:

[tex]\pi - z\sqrt{\frac{\pi(1-\pi)}{n}} = 0.55 - 1.96\sqrt{\frac{0.55*0.45}{400}} = 0.5[/tex]

Thus the correct answer is given by option c.

2) If licorice cost $6.59 a pound, how much would it cost to buy a quarter pound of licoric
(Hint: Convert the mixed fraction to an improper fraction or decimal and multiply by th
quantity required)

Answers

Answer:

$1.65

Step-by-step explanation:

[tex]6.59*.25=1.65[/tex]

or

[tex]6.59*\frac{1}{4} =1.65[/tex]

Question
Elvira and Aletheia live 3.2 miles apart on the same street. They are in a study group that meets at a coffee shop between their houses. It took Elvira 1/2 hour and Aletheia 2/3 hour to walk to the coffee shop. Find both women's walking speeds.

Answers

Missing from the question

Aletheia's speed is 0.6 miles per hour slower than Elvira's speed.

Answer:

[tex]s_E = 3.0[/tex]

[tex]s_A = 2.4[/tex]

Step-by-step explanation:

Given

[tex]d = 3.2m[/tex] -- distance

[tex]t_E = 1/2[/tex] --- Elvira time

[tex]t_A = 2/3[/tex] --- Aletheia time

[tex]s_E - s_A = 0.6[/tex] --- the relationship between their speeds

Required

Their walking speed

Distance (d) is calculated as:

[tex]d = speed * time[/tex]

For Elvira, we have:

[tex]d_E = s_E * 1/2[/tex]

For Aletheia, we have:

[tex]d_A = s_A * 2/3[/tex]

So, we have:

[tex]d_E + d_A = d[/tex] --- total distance

This gives:

[tex]s_E * 1/2 + s_A * 2/3 = 3.2[/tex]

Recall that:

[tex]s_E - s_A = 0.6[/tex]

Make sE the subject

[tex]s_E = 0.6+s_A[/tex]

Substitute [tex]s_E = 0.6+s_A[/tex] in [tex]s_E * 1/2 + s_A * 2/3 = 3.2[/tex]

[tex](0.6+s_A)* 1/2 + s_A * 2/3 = 3.2[/tex]

[tex]0.3+1/2s_A + 2/3s_A = 3.2[/tex]

Collect like terms

[tex]1/2s_A + 2/3s_A = 3.2-0.3[/tex]

[tex]1/2s_A + 2/3s_A = 2.9[/tex]

Express all as decimal

[tex]0.5s_A + 0.7s_A= 2.9[/tex]

[tex]1.2s_A= 2.9[/tex]

Divide both sides by 1.2

[tex]s_A = 2.4[/tex]

Recall that:

[tex]s_E = 0.6+s_A[/tex]

So, we have:

[tex]s_E = 0.6+2.4[/tex]

[tex]s_E = 3.0[/tex]  

Find the exact values of the six trigonometric functions at “a” given cos(2a) = - 4/5 and a is
in the 2nd quadrant.

Answers

If a is in the second quadrant, then cos(a) < 0 and sin(a) > 0.

Recall the double angle identity for cosine:

cos(2a) = 2 cos²(a) - 1 = 1 - 2 sin²(a)

It follows that

2 cos²(a) - 1 = -4/5   ==>   cos²(a) = 1/10   ==>   cos(a) = -1/√10

1 - 2 sin²(a) = -4/5   ==>   sin²(a) = 9/10   ==>   sin(a) = 3/√10

Then we find

1/cos(a) = sec(a) = -√10

1/sin(a) = csc(a) = √10/3

sin(a)/cos(a) = tan(a) = -3

1/tan(a) = cot(a) = -1/3

Which of the following is the vertical asymptote for the graph below?

Answers

Answer:

C

Step-by-step explanation:

Vertical asymptotes are always in the form x = ?

If you look at the dotted line, it lands on 2.  Because it's a vertical line, the asymptote is going to be x = 2

The ratio of yes votes to no votes was 3 to 4. If there where 2721 yes votes how many no votes where there

Answers

Answer:

3628

Step-by-step explanation:

Ratio of   yes votes:no votes = 3:4  

Yes votes = 2721,

Now,

[tex]\frac{2721}{3} = 907[/tex]

so, [tex]907 * 4 = 3628[/tex]

So, there were 3628 no votes.

3628.

Step-by-step explanation:

The ratio of   yes votes: to no votes = 3:4  

Yes votes = 2721,

Now,

2721/3 = 907

So 907 × 4 = 3628.

So, there were 3628 no votes.

How do I calculate a ratio?

Ratios compare two numbers, usually by dividing them. If you are comparing one data point (A) to another data point (B), your formula would be A/B. This means you are dividing information A by information B. For example, if A is five and B is 10, your ratio will be 5/10.

Learn more about ratios at

https://brainly.com/question/2328454

#SPJ2

Five friends each spent the same amount of money, x, on school supplies. They expected to spend a total of $130, but the actual
amount spent differed from the expected amount by $15.
Which equation represents the situation? What are the possible amounts that each friend spent?
O
13 - 130 = 15
The amount that each friend spent is $115 or $145.
O
151 - 130 = 15
The amount that each friend spent is $23 or $29.
O
151 + 15 = 130
The amount that each friend spent is $31 or $38.
O
(15.3 – 1300 = 5
The amount that each friend spent is $8 or $9.

Answers

Answer:

its the second option

Step-by-step explanation:

hope this helps

Answer: 5x-130=15

Step-by-step explanation:

edmentum

Identify the missing terms in the given arithmetic sequence.
5, ?, ?, ?, -9.

Answers

Answer:

the whole sequence is

5, 1 1/2, -2, -5 1/2, -9

the missing elements were

1 1/2, -2, -5 1/2

Step-by-step explanation:

an arithmetic sequence means that there is always constant added or subtracted from the previous term to create the next term.

we know, we need to go 4 steps or terms to get from 5 to -9.

5 + c = ?

5 + c + c = ??

5 + c + c + c = ???

5 + c + c + c + c = -9

5 + 4c = -9

4c = - 14

c = -14/4 = -7/2

so, we get

? = 5 + -7/2 = 10/2 - 7/2 = 3/2 = 1 1/2

?? = 3/2 - 7/2 = -4/2 = -2

??? = -4/2 - 7/2 = -11/2 = -5 1/2

and the next step is -11/2 - 7/2 = -18/2 = -9

correct.

Answer:     1.5, −2, −5.5

Step-by-step explanation:

Please help out would really appreciate it

Answers

Answer:

Step-by-step explanation:

1. Apply the Pythagoras theorem to determine the value of x, we have;

[tex]/Hyp/^{2}[/tex] = [tex]/Adj 1/^{2}[/tex] + [tex]/Adj 2/^{2}[/tex]

[tex]x^{2}[/tex] = [tex]15^{2}[/tex] + [tex]8^{2}[/tex]

   = 289

x = [tex]\sqrt{289}[/tex]

x = 17

2. Trigonometric ratios of <D.

i. Sin <D = [tex]\frac{opposite}{hypotenuse}[/tex]

              =  [tex]\frac{8}{17}[/tex]

ii. Cos <D = [tex]\frac{Adjacent}{Hypotenuse}[/tex]

                = [tex]\frac{15}{17}[/tex]

iii. Tan <D = [tex]\frac{Opposite}{Adjacent}[/tex]

                 = [tex]\frac{8}{15}[/tex]

3. Trigonometric ratios of <F.

i. Sin <F = [tex]\frac{opposite}{hypotenuse}[/tex]

             = [tex]\frac{15}{17}[/tex]

ii. Cos <F = [tex]\frac{Adjacent}{Hypotenuse}[/tex]

              =  [tex]\frac{8}{17}[/tex]

iii. Tan <F = [tex]\frac{Opposite}{Adjacent}[/tex]

               = [tex]\frac{15}{8}[/tex]

Please help! Thank you :)

Answers

Answer:

FE→

Step-by-step explanation:

You start with the endpoint and go in the direction of the arrow

FE

It will have an arrow on top with an endpoint on the left and an arrow going in the same direction to indicate it is a ray→

Other Questions
As long as you resist your role in creating the outcomes and experiences in your life, you will fallfar short of your potential. What are the procedures use in installing a sound card Is 1 2/6 a rational number? o que funciona melhor no nosso corpo com a pratica de atividade fsica? Price of benis house increased from 100,000 to 200,000 in the last decade. Cost of living quadrupled ( increased 4 times as much). What can you say about Benis house? Directions: Identify the nouns and pronouns in each sentence.1. Genghis Khan was a ruler of the Mongols in the early 1200s. wrote the terms below.8, 4, 0, 4, 8, 12What do these terms represent? an arithmetic seriesan arithmetic sequencea geometric seriesa geometric sequence Evaluate (f *g)(x) if f(x) = 2x^2 and g(x) = 3x 2. What opportunities to present reading practices skills faith is 16 years older than precoius in 10 years time. faith will be double Precious age. How old is precious? 2. Sunshine of Your Love is one of the best-known songs by Cream. What is the basis of this song? What influences can be heard in this piece? Does the bands technical prowess add or detract from the performance? simplify 2x2a^2 x2a^2simplify36a^3 x 1over 4a^2express 64 in index formsimplify 5^2 x m^2 and pls also explain briefly how u got the answer Rubin grew 9 tomatoes with 6 seed packs. How many seed packs does Rubin need to have a total of 21 tomatoes in his garden? what is the meaning of Neccesity A worker with a wheelbarrow filled with bricks starts at a point 50 m from theentrance to a construction site. The worker pushes the wheelbarrow away from theentrance at a speed of 1 m/s for 10 s, stops for 5 s to unload, and then moves backtoward the entrance at a speed of 2 m/s for 20 s. Draw a distance-time graph. A solid oblique pyramid has a square base with edges measuring x cm. The height of the pyramid is (x + 2) cm.A solid oblique pyramid has a square base with edges measuring x centimeters. The height is (x + 2) centimeters.Which expression represents the volume of the pyramid?StartFraction x cubed + 2 x squared Over 3 EndFraction cm3StartFraction x squared + 2 x squared Over 2 EndFraction cm3StartFraction x cubed Over 3 EndFraction cm3StartFraction x cubed + 2 x squared Over 2 EndFraction cm3 Answer this please~!!!! General Snacks is a typical firm in a market characterized by the model of monopolistic competition. Initially, the market is initially in long-run equilibrium, and then there is an increase in demand for snacks. We expect that: Group of answer choices in the long run, new firms will enter the market. there will be a short-run increase in the number of firms, but in the long run the number of firms will return to the original level. firms will leave the market in the long run. firms will shut down, but they will not leave the industry in the long run. Please help me! I will give brainly!!! Question,Your friend says that because ofthe order of operations, theexpressions are equivalent. Is yourfriend correct?16 (20 4) + 3 or 16 20 4 + 3 Yes or No Question,Which expressions have a valueof 8?A) 6 3 8B) 24 4 + 2C) 18 50 10 5D) 24 8 2 + 4E) (14 8 5) 8F) 4 4 18 2Check a letter Question,Three pumpkins are catapulted at acontest. * Pumpkin A travels 0.98 kilometer.* Pumpkin B travels 0.45 kilometer farther than Pumpkin A.* Pumpkin B travels 0.09 kilometerfarther than Pumpkin C. How much farther doesPumpkin C travel than Pumpkin A? A company is reviewing tornado damage claims under a farm insurance policy. Let X be the portion of a claim representing damage to the house and let Y be the portion of the same claim representing damage to the rest of the property. The joint density function of X and Y is Determine the probability that the portion of a claim representing damage to the house is less than 0.2.